Your Daily Equation | Live Q&A with Brian Greene

Video Statistics and Information

Video
Captions Word Cloud
Reddit Comments
Captions
everyone it is that time of the week again for our live conversation where as part of your day of the equation I'm happy to cover anything that is equation relating and of course since equations are related to everything that means everything is game so truly feel free to ask me about anything that's on your mind you know me if I don't know the answer I'm straightforward I tell you if I have something to say on the subject I'm happy to offer it and let's begin as we sometimes do and I see Luis has already noted that he is in our conversation from Mexico but I do like to see the various places where everybody is from so if you don't mind just sort of saying where you're from I would love to see that and then as those countries roll by I will be all too happy to start finding the first questions and get moving on things all right so what do we have here what am I going to begin with uh there's one question I really want to begin with and I I didn't have a a chance on YouTube to locate who it was that asked it but somebody asked in one of the last two sessions about what it means when I say when people say that time is curved right in the general theory of relativity in special relativity - of course we know that space and time are all together they're blended into this uniform continuum that we call space-time so if we learn that space is curved than the general theory of relativity then we must know the time is curved - but you know we all have an image for what curved spaces but what does curved time mean and the answer to that is less mysterious than it sounds it's not like you have a a Dali esque like clock that sort of warp time in some literal spatial way of course a clock is warped in space it's not itself warped in time but when we say that time is warped we simply mean that the rate at which time passes at different locations need not be the same and in particular what we learned in the general theory of relativity is that if you have a massive body that exerts the force of gravity or curves the environment around it the space-time environment what that means is the closer you get to that body the slower clocks tick so if you had a clock say that is at the top of the Empire State Building you have another clock on the ground floor there at 34th and Fifth Avenue would whatever the right address is I think that's correct then the clock at the top of the Empire State Building will tick off time a little more quickly than the clock at the bottom of the Empire State Building so time has warped in that sense you know if you have a ruler that's worked then different parts of the ruler will measure different distances between the same two locations right that's what a warped non-uniform ruler would do if you have time that's warped then clocks will not measure the same elapsed time between two events by virtue of this warped or curved nature of time now for the example of the Empire State Building you'd say well why don't I know about that the reason of course you all absolutely already know as well the amount of time warpage time curvature between the top and the bottom of the Empire State Building is tiny tiny tiny fraction of a second we just don't notice it but if you took that same example and you put the Empire State Building at the edge of a black hole then the clock near the edge of the black hole closer to the source of gravity would tick off time far more slowly than the clock farther away and that would imprint be a measurable time difference try to think of as mentioned before in interstellar you saw that example directly as the astronauts that went down to the planet that was closer to the black hole than the crew member who stayed up in the ship they just spent a couple hours in that environment but time is ticking off so slowly for them so quickly compared to the clock on the mothership that when they returned to the ship their fellow crew member had aged like 20 years where they had only aged a couple of hours so that's what we mean by curved time all right moving onward what else do we have here so salim yogi says dr. green thank you for amazing words is it scientifically correct to say that gravity is due to a field property of space-time or is it just pure objective geometric property of space-time and it is a very it's a very good question because we physicists freely go back and forth between the language of a gravitational field and the language of the geometry of space-time and I would encourage you to feel comfortable going back and forth between those languages as well in fact you should feel comfortable using the standard Newtonian language as well where you talk about a force of gravity one object pulling on another because depending on the problem at hand one or another language can give you the greatest insight Einstein's general theory of relativity language is indeed one in which we think about gravity as nothing but the curvature of space-time so why do objects move in interesting trajectories under the old Newtonian language of the force of gravity Einstein's answer they move in these interesting trajectories these orbits because they are sliding along these grooves these indentations these valleys in the geometrical structure of space-time so there's a sentence where I used the Newtonian gravitational force language which is also the Newtonian gravitational field language and also the Einsteinian geometrical language for the shape of space-time and so freely move in between those languages as befits whatever situation you are in now let me just give one example and I'm gonna in your daily equation of next week boy it is really windy here I don't know if you guys can hear this but in a moment I made me to close down the windows even though it's quite hot in here but next week in your daily equation I'll make this point as well in the Newtonian description right the apocryphal or perhaps real story has it that this apple fell from the tree and it hit Newton in the head and the Einstein game description in terms of the geometry of space-time is actually quite different because the freely falling objects in Einstein's view they are the ones that set the standard of rest and therefore in an Einstein in description the Apple when it's in freefall after it detaches from the branch it's just sort of hovering there in space-time and it's Newton's head that comes up and slams into the Apple because Newton is not in a freely falling frame sitting on the surface of the earth the Earth's surface is getting in the way of Newton's freefall Newton can't fall through the rock of the surface of the earth even though his body would like to so that actually helps us understand the ancient puzzle that you all know the answer to but you know why is it that two different objects fall at the same rate regardless of their mass I could try to do this but I'll make a mess of it so a bit or not but you know what I mean you take two objects you know I don't want to break the Apple pencil then I won't be able to do your daily equation but you know I let them go and they both fall together why is that according to Newton it's because there's equality between gravitational mass and inertial mass which means they both cancel out of the F equals MA equation when the force is equal to M over R squared times G that's equal to M times a the ends on both sides cancel so it's only the big mass of the earth that really determines the acceleration of each of these objects they predict fall together but that's a mystery why is gravity facial mass equal to inertial mass Einsteins answer is completely different Einstein says when you let go of these two objects I'm not gonna do it again but when you let go of them they're in freefall from your perspective which Einstein says sets the standard of rest now imagine therefore that they are at rest in space-time and it's the earth that's rushing up to them so why do they hit the earth at the same moment Einstein's answer is very straightforward they hit the earth at the same moment because they don't move it's just the earth that comes up and meets them and the earth has a singular body when it moves up to their location and hits them and of course it hits them at the same moment they were just sitting there waiting for the Earth's arrival so that's the geometrical einsteinium view and in that case it does give us insight that otherwise would be hard to glean okay so what else do we have alena says big fan of your videos thank you Alena I appreciate that um Abu Bakar Faruk we greetings professor green watching you from Pakistan wow that's great to hear what is your view regarding the idea that the entire universe is inside a black hole well it's an interesting question first of all talking about the entire universe inside a black hole is a bit extreme but but what I would say is it is possible that at least according to the traditional general relativistic description of black holes I'm not going to modify it with modern or more recent conjectures speculations about firewalls and all that sort of good stuff so let's just act as though Einstein had the important word on black holes which may not be the case but if that's the if that's true then right now it's quite conceivable that the earth just passed across the edge of a black hole that we are all now inside of a black hole why is that the edge of a black hole in Einstein's description is not some special location in space it's just an ordinary chunk of space that has the usual property that once you pass that location the event horizon you are no longer able to get out but you don't have any direct physical reaction to crossing over the event horizon of a black hole and so right now everything you know as we look around the world we could all be within a black hole and will only really recognize that later on when we get closer and closer to the center of the black hole because then depending on our precise distance from the black hole different parts of the earth different parts of our body will feel a different force if we are going feet first toward the center of a black hole our feet will experience a greater pull than our head our body will be stretched spaghettified is the technical term that we use for this process until ultimately we're pulled apart into our constituent particles that will all rain down on the singularity whatever that is the center of the black hole we don't really understand what goes on there so in that sense we could all be within a black hole right now I'm not exactly sure how to describe that if you want to truly think of the entire universe but you know any large chunk of the universe could follow exactly the same process that I just described for the earth and therefore in that sense we could be within a black hole um Nikhil Taha Lonnie says hello professor iam a computer science student in India boy a lot of Pakistan India - it's fantastic recently developed interest in physics because of you oh that's great to hear it really is great to hear it's so inspiring to hear that you know these little videos that that I created in this little room here other things that we do actually reach out and and affect people so thank you for telling me about that question is can you guide me if I can do both together namely computer science and physics and where I should start and the answer is out absolutely number one the distinctions between computer science and physics have certainly been blurred in in the last few years in a substantial way because one of the dominant interesting problems that people who have been focusing their attention on quantum mechanics for a long time that they're focusing on is quantum computing now to really see whether we can inject our understanding of quantum physics into a new kind of computational procedure now look of course every computer in some sense is a quantum computer because the integrated circuit only exists because we can manipulate the motion of tiny particles like electrons and we need quantum mechanics to understand how to do that but quantum computing picks that into a radically different direction not just the technological innards of a computer to be governed by quantum physics but rather the computational process itself making use of the full might the full power of quantum mechanics roughly speaking if you buy into the many-worlds interpretation of quantum mechanics it's as if we're trying to build computers that will do parallel processing of certain kinds of computational procedures across the many universes that quantum reality spans make use of all of those universes to simultaneously crack apart various parts of an interesting calculation at the end find a matter of pulling out the solution that we are looking for from this broad swath of parallel computational procedures so I don't know if that kind of blending of physics and computer science fires you up to kill but that certainly is an arena in which you would be well posed to make a contribution by carrying on with your studies in computer science but also building up your understanding of physics with all the standard things classical mechanics quantum mechanics thermodynamics you need all of that and some of that you can learn on your own so I would look at you know wherever you are the course of study that's available to see about blending these but it's not as though you're saying to me you know I want to I want to dance in the Bolshoi Ballet and I also want to be the next Einstein not impossible and I would actually I would actually encourage you if if that was your dream but what you're saying is easily more easily attainable than that particular goal that particular dream because computer science and physics are are no longer as far apart as perhaps your studies might suggest that they are ok what else do we have there's an interesting question here oh here's one from China you ping Jiang hi professor a big fan from China thank you you ping if I move close to the speed of light relative to you but from my perspective you are the one who is moving fast so in the end who is getting older and yeah you ping that is a puzzle it is one that people have kicked around for a long time we think we understand it quite well which is that the notion of Hugh who is getting older is a question that needs to be framed with greater precision in order to really answer your question and the real question is if you and I get back together and we compare our ages either visibly or more precisely with our watches on whose watch will more time have elapsed and if it's the case that I say I'm not experiencing any say acceleration during this entire journey right this entire journey I'm just sort of sitting around and I never feel a force on my body I never feel a change in my speed but you in order to come back to me must feel that change let's say you go out in space you've got to turn around and come back that turnaround is an acceleration you will feel it it is a change in your velocity a change in the speed and direction of your motion and that's what breaks the symmetry that otherwise would be quite confusing and because of that break and symmetry you feel the acceleration but I do not then it turns out that you can establish that you are the one that is going to be younger on the return I will be the one who is old it is your watch that would be ticking off time more slowly now you might say that feels like we're relying too much on acceleration to address the issue and indeed there are other versions of the story you could go out into space and rather than literally you turn around imagine that you've worked out with a friend of the or as a partner in crime that they are heading toward me and they set out on the journey in just the right way that you and your friend pass each other and you're able to hand off your watch to your friend and your friend brings that watch back here so no one's accelerated in this case you never turned around you came back your friend didn't accelerate they were always traveling toward me what happens in that circumstance and the answer is a little bit different during the handoff between you and your friend that is a change in frame of reference which actually changes your perspective on what events are happening simultaneously at the same moment if you take that into account there's a jump in time between what your friend what you said was happening at the same moment and what your friend says is happening at the same moment and that also comes in the exact same conclusion if you want to see this done fully and visually obviously there's just so much I can do in explaining things on the fly right here but I have a whole course on the special theory of relativity there's a math version of it there's a visual version of it you can look at these courses take these courses for free at world science u.com that website is being relaunched refreshed I think it's still available right now but if it's not within a week or two it will be available and toward the end of that course they're like 44 modules a lot of content in there but I show you visually what it would look like in sort of any scenario of the twin paradox that would come to mind and so the short answer is I will be the one who is older if I didn't accelerate the long answer is go check out all the detail unreal sign - I think I think you would enjoy doing that and that goes for any of you who want to see special relativity done in a highly visual as well as mathematical way okay what do we have here hello sir a seat - says you were my inspiration thank you how is how in string theory is it that vibrations give rise to new kinds of particles and yeah that is the basic idea in string theory and let me just explain the essence of it so in string theory we imagine at least in the most vanilla bread and butter approach to string theory people gone in many different directions with these ideas let me just give you the the dominant way that people have thought about things for a long time if you were to take an elementary particle like an electron or a cork and somehow we're able to magnify it to an extent that you could see its most deep inner structure you'd find that each of those particles is composed of a vibrating filament a one-dimensional vibrating filament and you might say how could particles as different as an electron the cork they've got different electric charges they respond to the strong nuclear force in different ways how is it that a string can distinguish them and and here's the idea and I'll focus on mass is that's the easiest example easiest quality for me to talk about the mass of a particle in string theory comes from its energy right Einstein taught us e equals MC squared the most famous of all equations the equation I started with a month ago in this series of your daily equation let's turn that equation around actually turn it around in the way that einstein himself wrote it in his September 1905 paper he didn't write equals MC squared back these different symbols but put back to the side instead he wrote M equals E over C squared M equals E over C squared so mass comes from energy divided by C squared so if you want to know the mass of a part of that emerges from a vibrating string you find the energy of the vibrating string divide it by C squared so how do you get two particles of different masses you get two particles of different masses by having two strings vibrating with different energies one is vibrating slowly if I can use ordinary language one is vibrating more vigorously faster greater energy greater mass so that's C that's the way that the one and the same ingredient the same vibrating filament is not like you've got different filaments of the same species of filament but if they vibrate in different ways you get different masses now to describe different electric charges is a little bit more subtle that would take me pretty far afield but it's qualitatively no different from that it's just there are more exotic qualities of a vibrating string that correspond to charge and if it's vibrating in one manner in those qualities that will yield one charge vibrating differently will yield a different charge okay what do we have here Ali unis cyan as how should we study to be a good physicist please specify brian is very important to make sure look the most important thing if you're asking me when you're in the learning phase of physics is that you don't get distracted by things that are happening at the frontier of research and think that you can jump right to the edge of understanding without slogging through all of the basic foundational material I can't tell you the number of students that come to me and they just say hey I'm I'm ready I just want to work on string theory I want to work on black holes or I want to work on the Big Bang I mean it's exciting for me to hear the interest in working on those subjects but the fact of the matter is if you don't understand classical mechanics and quantum mechanics and differential equations and thermodynamics and statistical mechanics and special relativity and general tivity and quantum field theory and on it goes the traditional graduate and graduate curriculum if you don't have that solidly under your belt it's gonna be very hard for you to contribute so although it may feel that that materials old and you don't really need to know that material the fact of the matter is physics is a very hierarchical vertical subject the things that we work on today rest upon the things that we learned yesterday and by yesterday I mean everything from truly yesterday back to a few centuries ago so the answer is you just need to work day and night to learn the basic material and it's not just a matter of reading it it's a matter of doing problems and problems you need to gain a facility with putting the material into action so it's a lot of hard work but the payoff is significant number one you understand the materials so much better when you work through it every equation you understand every problem in the book you can solve but it then prepares you for bringing all that knowledge to bear on cutting-edge research and that's really the way to get there there's there's no shortcut unfortunately göran kohler asks how do you balance life and work well you should ask my wife and kids not well all the time sometimes I do it better than others but I kind of work all the time I kind of work seven days a week I don't come to Saturday and Sunday and say oh it's over I don't have to think about things until Monday and you know my family has come to accept that perhaps the reason is my wife and I both are the the folks who started this world Science Festival which is the rubric under which this your daily equation and many are the things sits so because we work together on this it doesn't always feel as though I am leaving the family conversation if I'm working on things in least in this regard my kids get a little bit tired of the fact that my wife and I both work a lot all the time so yes the balance is not is not always there but I certainly have found the ability more now than say 20 years ago to pull away from either doing a physics research project or writing a book or doing this kind of an activity to spend more time with kids and I have to tell you it's the most important the most vital thing to do and this pandemic only brings a home even further in the end what really matters what really matters are the people you spend time with and the people that you affect and the relationships that you have because in the end everything just evaporates into the void that doesn't mean it's not important to strive it's not important to understand the universe I'm driven to do that but in the end what really matters are are the relationships and it's hard to keep that in mind but it's it's vital to do so all right what do we have here cosmic slice I'm going to miss the World Science Festival this year I love volunteering at it yes I agree you know we're going to do a kind of World Science Festival this year we're not going to do it like all at once in a five-day period as we do when it's a live event we will roll out a series of programs over the coming months some of those programs we're going to will take place right in this space right here others we have other ideas up our sleeves for what they will look like but from that perspective unfortunately volunteering is not something that is relevant with the way the world is at the moment but the program's themselves they will be coming so at least that side of it will carry forward professor green Divya Raj asks what to do when your school intervals interferes in your education that is an encapsulation of a quote of the man himself Albert Einstein and I appreciate that reference Einstein said the only thing that interferes in his learning was his education you know but Einstein was Einstein Einstein was the kind of guy who could go off and read the Great's of physics you know read it read Maxwell's original works Keir cough Reid Lorentz you know Reid plank and and not only digest what they were talking about but be able to take their ideas to the next level unfortunately most of us are not Einstein what I'd love to be at the level of an Einstein yeah I really would how exciting to be able to revolution our understanding of things but you come to terms with the limited capacities that you have in any domain and for most of us that limited capacity means we do have to be educated most of us cannot just do the Einstein thing or do the Ramanuja thing where we can just learn on our own you know that would be great if we could and if it works for you fantastic but for most of us the education process doesn't interfere with our learning it's part of the learning I hope you guys are not distracted by my dog who seems to be barking outside the window that probably is because my dog Luna has a different perspective on learning and education and understanding it wants to chime in but you know Luna you want to say something get your own damn show this is my show okay so I'm just gonna say my view I'm learning in education no Luna if you have anything you want to say feel free to chime in again and I'll try to interpret for the audience okay so Muhammad Xun a ho asked dr. Brian in thermodynamics we have something called a closed system in which only energy can be exchanged not matter isn't it against mass energy equivalence well the the usual way that we do thermodynamics and I hope I'm interpreting your question correctly if not feel free to ask it again if I misinterpreted it in in thermodynamics there are two ways in which we formulate the theory when we apply the equations to a close to system we really mean that it's really close in the sense that it's not interacting with the environment around it all the stuff all the energy all the mass it's all staying within that environment the alternative is when we look at an open system and an open system freely can't exchange things with the environment now you're right in certain circumstances we only allow certain qualities to be exchanged but the real reason why we distinguish you being a closed system and an open system is for a closed system we can argue that the entropy will not never go down but is highly highly unlikely to go down whereas for an open system entropy can leave it can be expelled with the stuff that you are allowing to cross the border the open border if you will and therefore an open system and to be can go down and of course when I look at myself I am an open system in the sense that I'm constantly emitted emitting entropy to the surroundings heat waste going outward and that allows my entropy to not continually increase which would not be good for my health so the way I keep my health the way I keep myself intact is through my interactions with the environment allowing myself to be an open not a closed system and that openness is what allows my entropy to seemingly thwart the second law of thermodynamics but of course it doesn't because if you take account of the entropy that I released the environment then with that contribution the overall entropy of me and the surroundings does go up okay what else do we have here there's an entanglement that just went by me and I missed it here it is Oh Priya - why are particles entangled just by interacting and it's not enough that particles interact - to have the kind of entanglement that for instance I used in your daily equation in describing entangled that was for called maximal entanglement where I had these two particles one is spinning up and one was spinning down you remember how that goes and and that state of affairs will not generally be in force by simply two particles interacting the way you get those kinds of entangled particles is for instance you take like one example is you take a calcium atom and you excite the calcium atom maybe you shine some laser light on it as the calcium atom falls back down to the less excited state it releases energy in the form of photons and one way that it can release that energy is giving off back-to-back photons and those back-to-back photons will be analogous to the spin-up and spin-down electrons they will be maximally entangled they will have an overall zero angular momentum state even though each of them can have some spin angular momentum and those spins will be correlated in such a way that they mimic what I was describing with the entangle electrons so in that case it's not that the the photons interacted to become maximally entangled rather the calcium atom is the unique common starting point and when it reduces to its less excited state it emits these particles it creates these particles in this maximally entangled state Titan says god I love science that's an interesting sentence I love that god I love science I'm not sure how to to fully interpret it but it seems to have a good feel to it let's see what do we have here a su Susan van de ham Susan band ahem ah I got that aka primordial soup do you think there really is an end to smallness do I really think there's an end to smallness so I gathered what that question is you know the progression of the history of science is you got stuff macroscopic bulky stuff you look at it sufficiently powerful microscope or you dissect it you get two molecules you get two atoms atoms you get two electrons around the nucleus the neutrons the proton inside the neutrals approaches got the quarks if string theory is right and we don't know that it is you keep on going you get to the string as a question is does it keep on going or is there an end and we don't know the answer to that my guess is that there is an end I don't see any reason why this chapter of exploration needs to go on forever it could well be that the universe truly does have some fundamental indivisible uncuttable elementary constituents do we have those in hand it's possible that we do already it's quite possible that we do not however the motivation for thinking that this journey will go on forever is one of pattern the pattern is that we have always found smaller stuff inside of bigger stuff but I don't see any fundamental reason why that pattern will persist indefinitely so if you ask me for my guess and it really is just a guess my guess is that there are fundamental entities that will not admit a finer level of structure and there are fundamental laws that govern how those fundamental entities behave now it's interesting you know somebody the other day challenged me on my use of govern in a sentence like that do laws govern how the particles behave or do the particles just do what they want to do and we have these mathematical laws that seem to be pretty good at describing what they are doing and you know what's my view on that my my language certainly my language is I tend to use the word govern because I really do as I said I really do think that even if we don't have the fundamental laws in hand my view is that the universe is lawful it's an assumption but it's an assumption that seems to resonate well with everything that we have so far discovered about reality that there are laws and those laws are unbreakable and if you have unbreakable laws it feels to me it suggests to me it seems to me that you are justified in thinking of those laws as governing what happens because the particles cannot simply do whatever they want to do the particles only execute behaviors that are compatible with the fundamental governing laws so so that's my in some sense view of the fundamental nature of reality fundamental nature of the world there are some elementary things that can't be divided further there are some fundamental laws that are in control of the behavior of those entities the entities don't have a mind of their own in the sense of they can say well the fundamental law says X but I'm gonna do Y first of all that sentence doesn't really even make sense but more importantly it's not compatible with our understanding of how the particles that we do have control over actually behave so so yes the idea of laws governing stuff is the view of reality that I think is the right way of thinking about things and I prove that no can anyone prove otherwise you know the only data suggest that that is the right way of looking about things so it feels to me like it's a reasonable way of describing reality so fella super says hi Green greetings from Norway doesn't quantum entanglement break the principle and special relativity that simultaneity is relative it actually does not I understand your thinking on that you know if you have distant particles and they're very far apart and if you measure one and I say instantaneously the other snaps out of the haze of up and down I'm saying well that's simultaneous but what I really mean it's simultaneous from one perspective from one frame of reference let's imagine that these two particles are not moving relative to me this frame of reference that I am in then the action that I'm talking about is simultaneous in this frame but if if you fellows super are moving relative to me then you would tell a different story of what happened if you're moving relative to me in one direction you will say one particle snapped out of the haze first and then the other snapped out those events would not be simultaneous for the very reason that you said that simultaneity is relative and if you're moving in the other direction you will say the other particle snap out of the haze first and then the second particle snapped out subsequently so depending on your frame of reference your description of what happens in the measurement of an entangled pair will be different the point that I'm making is there is a frame of reference in which they happen simultaneously and that means and let me just stress this so that it can be no confusion here you might say well if one snaps out first and then the other why can't we describe it in terms of a signal from one to the other but bear in mind that if you have two particles snapping out simultaneously their so-called space like separated which means that even though the events may appear to have a chronology one happens before the other depending on your frame of reference you will never have a frame of reference in which there's enough time between one snapping out and the other for a signal to travel between them so everybody and his brother and his sister will agree that there's no way that a signal could go from one to the other to explain one snapping out and then the other some will say that they happen simultaneously others will say that they don't happen simultaneously but everyone will agree that you can't explain it with a signal traveling less than or equal to the speed of light going from after it snapped out alerting the other and then it snaps out that explanation everyone will agree that that explanation fails okay so Kella me neva says from a big fan in Brazil well thank you I'm more than happy if you just want to chime in with comments you don't have to only ask questions thank you Boogaloo ball edge of the wedge theorem asks what type of mathematics have you used but you where you were surprised that it is helpful in this case well yeah you know I give you one concrete example it goes back a number of years but it was among the most exciting years ago I was working on properties of the extra dimensions in string theory you know that string theory requires six or seven extra dimensions depending on how you how you count how you describe it and I became interested with a couple of colleagues David Morrison and Paul Aspinwall on whether the fabric of space might be able to rip to tear and to address this question we found that we had to invoke some fairly esoteric mathematics from algebraic geometry and from a fairly esoteric at least to me at the time way of representing geometrical surfaces that that naturally emerge from algebraic geometry and with that description we were able to get a sharp mathematical understanding of the fabric of space and were able to show that it can actually rip apart you may have read about this if you've looked at the elegant universe in I think it's chapter 11 or 10 or something like that it's too long ago now by the way I'm actually planning on coming out with a new version of the elegant universe for the one or two of you who may have read that book and it's going to have an update I'm not going to change the chapters that exist but I'm going to update the book bring it right up to the present so for the 25th anniversary which is in 2024 it's hard to believe is 25 years ago but for the 25th anniversary edition of that book I'm going to add some chapters so you can catch up on things that you might have read elsewhere which would be kind of fun but that's the answer to your question Boogaloo both edge of the wedge theorem that was an example where some pretty high-powered math and algebraic geometry turned out to be the right language for discussing the ripping of space campus cryptid writes hell yeah I'm not sure what Hellyeah refers to but hopefully it was something that I said that you were so excited about that you removed to to write that let's see what we have here is well I'm gonna go to this other list from Twitter for just half a second just so I mix it up a little bit what do we have here Nikita geico asks tell me if I'm wrong that most of the current mass and energy in the universe was condensed from the in photon field after a tremendous expansion so the total energy of the universe increased does this break the first law of thermodynamics of energy conservation and you're not mistaken Nikita in your description so let me just even put some numbers into the inflationary theory the inflationary theory shows that you can have a little tiny region of space roughly say 10 to the minus 26 10 to the minus 27 meters across so it's really tiny it can be filled with you know roughly uh a few ounces or a couple pounds worth of in photon or maybe I should talk about kilograms for this international audience so it could be filled with a vo grams or kilograms worth of the in photon field so not a lot there and as it swells from me for a pulse of push of the in photon field that region of space swells enormous Li as it swells the in photon field continues to fill that region of space so you may have begun with say kilogram of M photon field but if the size of the space grows by a huge factor then the amount of in photon field filling it also grows by a huge factor so then you might say well where does the energy come from because now you've got more because the region of space is incomparably larger and the answer that we typically give to that well there are two answers you could give one answer that's not really satisfying that you give is that the conservation of energy has to be carefully thought through when you apply it to the universe as a whole and whereas you can have local energy conservation turning it into a global statement can be challenging if you looked at the episode on no--there's theorem that we had and unless people looked at the episode unnotice theorem which makes me think that you actually overall don't really like the technical mathematical equations as much as you do the discussion of the big idea so tell me if you disagree with that because my temptation is to do more of the big ideas if it appeals to more of you but that's a sidelight in that episode in your daily equation I showed how you get global conservation laws from symmetry but that was applied to a system within the universe if you look at the universe as a whole and you think through the steps there's some subtleties that don't always go through depending on the precise shape the topology the geometry of the universe but anyway I didn't think that answer would be particularly satisfying so I just put that out there as an interesting point the dancer that I think you'll like better Nikita is the following you may recall that gravity can have and typically does have negative energy remember when you learned about gravitational potential energy in physics 1 what is the gravitational potential energy it goes like minus GM over R right if you have a mass M and that's a negative in front so when R goes to infinity say a little test particle would have zero potential energy but as it comes in it's in a gravitational field giving rise to a negative potential minus GMM over R second and the little guy over R the importance of that is the following a negative number I mean it can go arbitrarily negative so as the in photon field as its energy is going up and up and up filling more and more space the gravitational potential energy is going down and down ever more negative so there's a balancing that can happen there so it may look like there's more energy because this space is bigger and it's filled with the M photon field but if you account for the negative potential energy of gravity becoming ever more negative they cancel each other out allowing a kind of energy conservation still to hold so that I think is a more satisfying answer Makita if you're actually watching the live chime in on the live feed and I can get a sense of whether that addressed your question let me go back to the live YouTube chat and see what else we have here let's see let's see what we got here car Braun's asks about quantum biology and expresses some skepticism saying I smelled something funny but with a with a smiley face and no quantum biology is a real serious and significant field you know mostly we think about quantum mechanics is being confined to individual atoms and molecules and things that are pretty small but remarkably scientists have found that there are certain quantum processes that take place in certain biological systems that have a macroscopic manifestation I don't remember the details we did a program on this in the World Science Vessel so if you're interested just search world Science Vessel quantum biology but there are certain navigation systems in birds I believe in which scientists have traced the birds ability to navigate on certain trajectories to certain quantum interactions between molecules in their brain and say the magnetic field of the earth or things of that sort so quantum biology is hugely exciting because it suggests that nature has found clever ways of leveraging some of the spectacular qualities of quantum physics into adaptive processes adaptive behaviors and now first thought you say well why wouldn't nature do that quantum physics is real quantum physics affects how particles behave and if a species comes along and by chance finds a means of tapping into those quantum processes more directly than we would have thought based upon you know everyday experience with magnets at macroscopic things in the world around us and if there really was adaptive value to it then that species would have a better chance of having progeny and passing on to the next generation that very same ability to leverage quantum physics so I don't think it smells funny at all it smells kind of sweet and exciting so from that perspective it really is an exciting development to see quantum physics manifests in biological systems okay what do we have here back to this feed here so you Flo says professor green my name is Eugene and here's my question can a clock be defined in a different way than a cyclical motion device atomic clocks of course are a cyclical motion device and I'm always reluctant to say no because there's always some clever thing that you can come along with but it's hard for me to fully understand what a clock would be if in some way shape or form it wasn't relying in some kind of cyclical process because what we do to talk about the passage of time is we count the number of processes that go by between the beginning the start of the event and the end of the event so we want a cyclical process and we want to be stable cyclical process so we can say hey it took ten of those processes for so-and-so to drink that glass of water or it took fifty nine of those cyclical processes for so-and-so to solve that math problem now we often call those cyclical processes by seconds if we tuned them in the right way but it's hard for me to really understand what we would mean by the passage of time in terms of a quantitative measure if we didn't have some kind of cyclical process that was stable so he could depend upon the cycles being identical thereby lasting the same duration so look no doubt there's something that someone's going to come back to me on with that but it's pretty profoundly woven into our thinking about time that we measure it with a device that has some cyclical process embedded within it okay okay what else do we have here dhama dark posad s asks is there a possibility for math to be different in a multiverse if it exists and sure look it depends on how how you interpret mathematics but actually based on either interpretation there is a version of your question that I would answer in the affirmative so one way of interpreting your question is this math twosome is nothing but a language it's a human invented language just like English just like Hindi just like Swedish whatever these are all just languages that emerged by virtue of the human brain finding it useful to codify certain patterns in natural language the patterns that we codify are the typical things that we encounter in daily life day to day life in mathematics the kinds of things that we quantify are more specific to particular processes that happen in the world you know you bring two coconuts - your friend someone else has one coconut you put the you got three coconuts there's a pattern there and of course we take those patterns as the brain matures and refines and describe ever more esoteric patterns less and less available to everyday experience but if you believe as some do that whether it's natural language or mathematics it is simply a human invented construct language then of course if there's a multiverse it's possible that other members in other universes will have different mathematics in fact you don't have to go to a multiverse it could be another planets forget about the multiverse and other planets those living systems they may have codified patterns in a different way or they may have found that the kinds of patterns that we find to be useful the seasonal motion of the planets around the Sun helps us know when to plant crops if on that other world maybe they live beneath the surface they don't see the Sun or a star the notion of orbiting and orbital periods just has no pull no value in their culture the mathematics that they come up with may be quite different from the mathematics that we come up with maybe it's not even mathematics at all maybe it's a whole different kind of way of engaging with the world and mathematics may not be anything that they've ever invented or paid attention to now to contrast that there are some people think that mathematics is not invented it's not a language mathematics is just truly stitched within the fibers of reality and we physical beings are nothing but an incarnation of the underlying mathematical equations that goes even further than anything that I really feel comfortable espousing I'm not ready to say that math is so fundamental that any mathematical equation is relevant to the physical world there are some who do if you want to read about them at for next tegmark has a whole book on that I think it goes a little too far for my taste but if that were the case then you might say could other places have different mathematics well they could have found different mathematics than we have found but it's all part of one grand collection of ideas that we call mathematics and we may have found one little island of that territory of math others may find other islands but we would all be part of the same overarching fundamental structure the mathematical reality that this perspective puts forward so two views each come to the conclusion that when we talk to the aliens their math may be different from ours every reason suspect that in fact it might be see of s72 men as how big is the unobservable universe well good the observable universe has a finite size it's simply how far away objects can now be and yet their light would have had time say since the Big Bang to traverse our separation and reach our telescopes or reach our eyes and that number is about 45 billion light years in each and every direction beyond that is the unobservable or the yet to be observed depending on how the universe expands universe how far does the unobserved part of the universe go on the answer is we don't know it could go on forever it could be infinite space might go on infinitely far which is so easy to say and so difficult to really understand infinitely far it never stops like where did that come from who provided that amount of real estate I mean these kinds of questions come to mind because we live in a finite environment everything may encounter is finite it's so hard to wrap your mind around something that is truly infinite and who knows maybe it's not truly infinite it's a possibility but nobody knows for sure it could be that you travel out into space officially far and you hit a wall maybe there really is an end out there and that's not so crazy there's a version of string theory in which you imagine the extra dimensions have a particular shape and one shape that people have discussed amounts to having to brick walls on either ends of this universe they might say what's on the other side empty space no nothing no reality is on the other side it's hard to picture everything that has a border right I mean this box you know my my air pots this box it has a border and outside the border there's more space but imagine that the border of the universe is the end of reality not emptiness not space that's yet to be inhabited nothing beyond the end that's a possibility - or it could be this face wraps in on itself right you know and I have my basketball here any longer but you know it could be that space is curved you go out in space in one direction you go far enough and ultimately without even turning around you wind up returning to your starting point from behind your initial location that's what happens on planet Earth if I walk out in that direction I go far enough I will circle around I'll circumnavigate the earth you perhaps conservative circles the right shape which would mean that it's finite it does not go on forever and yet there's no wall that you encounter it's simply the shape allows it to have the property of finiteness without an edge without a boundary so yeah it's a great question it's a tough one that's the current state of the art understanding Christopher asked dr. Greene which is your favorite my favorite what Christopher you must have been referring to something I said at that moment but I have no idea so Chris if you want to chime back in I'd be happy to tell you what my favorite is but without any noun there it's hard for me to answer ah little big von Beethoven is back so good to see you again Ludwig von Beethoven it's the name that always sticks with me because I am a great fan of the Ludwig von Beethoven but anyone who has his name I assume as a fan as well oh I see confirm the World Science Festival is sponsored by Apple we're not sponsored by Apple I'd be happy if we were sponsored by Apple I'd happily take their support to create all sorts of cool content so if you guys want to lobby Apple to support us I'd be all too happy but when I pick up any of my gear is simply because it is around me there is zero product placement there is ZERO monetization happening on any of the stuff that we are doing this is all just to bring these ideas out there no motivation beyond that ok I've been told by Siavash that I butchered the name sorry and there's no way maybe you could spell it out phonetically for me next time and do my best okay so we have rygg I asked dr. Greene can you go into more about the difference between the 50% chance of spin with quantum entanglement right versus the 55% theorized by Einstein where did the 5% go ah good question good question so remember how this went actually I don't have to take a sip of water is this this question is good it's gonna take me a moment to answer so remember I'm Stein's perspective he says he says come on quantum mechanics people can you really believe that if you have two widely separated objects each in a fuzzy statistical mixture going up and down that if you measure one that it can have an impact on the other arbitrarily far apart arbitrarily distant from the first he says come on you can't stably believe that and he says I've got a better way of explaining what's really going on here Einstein says look your quantum math says this particle is in a fuzzy mixture 50% up 50% down but that's not what's really going on that's merely a limitation of the quantum mathematics to describe the true reality if you measure that particle and you find it up it always was up the math might have said it was doing this but it wasn't really doing that at all it's a limitation of your knowledge of reality it was up all the time when you measured it you simply found what was already there similarly if this one you measure it you find it down it was always down so there's no spooky action because when you measure this one and find it up it was up when you measure this one you find it down it was down from the get-go always were in this complementary pair of up and down john bell comes along and says well Einstein couldn't talk to him directly by this point Einstein had passed away but Bell in essence says well Einstein that's an interesting suggestion the particles always have these definite features even though quantum mechanics is saying that they don't is there a consequence of that at first sight people thought no remember I think I mentioned that Wolfgang Pauli attributed the way of thinking about the world to being no better than trying to count angels dancing on the head of the pen it's as if I again the example I use with you guys we use it again it's as if I say to you hey guys my hair color is actually in a fuzzy mixture of pink and blue when you're not looking but when you look it snaps to one definite gray or white whatever the right description of my hair color may be how can you ever test that was in a mixture without measuring but everybody says once you measure you find a definite result so it seems like it's untestable now to your question about the 55% reigai re guy john bell shows that if you are measuring the particles in a more clever way not just along one spin axis but along three spin axes he finds that there is a measurable consequence to Einstein's perspective that the particles have a definite spin when you're not actually even measuring them he finds that these detectors if you do random measurements of the particles on a great many runs of the experiment they should disagree the particle should be up and down at least 55 percent of the time that's a consequence of Einstein's vision that there's no spooky action and that therefore particles have definite features that's testable you do the experiment and you don't get you don't get Einsteins prediction you find 50% of the time they are found to be in these oppositely oriented pairs so where did the 5 percent go it was not ever there Einstein's prediction was wrong Einstein didn't even know it made a prediction it took John Bell to figure that out but in essence Einsteins vision of the world no spookiness and therefore definite features is ruled out what does that mean well Einsteins vision no spookiness there for definite features that's wrong therefore there is spookiness there can be spookiness and definite features but there has to be long-range connection in this way of thinking about things that's the answer the 5% was an incorrect prediction the 55% therefore was an incorrect prediction 50% is the right prediction and it's the quantum mechanical prediction all right I hope that clarifies there where I got asked again it's I didn't convince you of that so what else do we have should I look maybe on a on a Twitter thingy what do we have over here hemant streak ass hello sir can you help us imagine non Euclidean geometry of space-time very hard I can help you understand non-euclidean geometry of space but I know that's not what you're asking nan you could add geometry space is just non flat surfaces that's the 2-dimensional version and the surface of a basketball is a really good example of a non Euclidean a sense of non flat surface but to bring time into the story it's harder to picture mentally you know the there the diagnostic that I gave in the last your daily equation is I wonder if I can even bring my iPad up just to remind you of the diagnostic I'm gonna try to do this I hope I don't cut everybody off so did that work am I on this thing here mmm-hmm I'm trying to share my screen with you guys let's see if this works if it doesn't work then I'll go back to just describing things in words but I think this is going to work and let me bring up a notability thingy here so hopefully that's working with us right now and you may recall that I gave you a diagnostic for when a shape is curved and the diagnostic is you take a vector at some location in that space and what you do is you move that vector around so I'm gonna do that here I'm gonna take a copy of this guy put them over here paste it and now I'm gonna move it and notice as I move this vector around on the plane regardless of what path I follow when I get back to my starting point the new vector is pointing in exactly the same direction as the original vector on a curved surface like a basketball in fact I used a basketball last time and I was describing it that will not be the case you don't return to your starting point the vector instead points in a different direction why don't I bring that up I bring it up because imagine you're in space time and in space time you can carry various things around you can for instance carry around a clock and as you carry around this clock the when you return to your starting point will the reading on your clock agree with the clock that stayed at your starting location and the answer to that in general for a curved space-time will be no as you Paulo transported around in the language of the field if not it does not return to its original configuration so curvature for a basketball that makes sense curvature of space-time harder to envision but it's the same kind of diagnostic tool fundamentally it's the same mathematics all right eric w asks you mentioned working all the time no interesting hobbies well over time I've sort of found that my hobbies have integrated inserted themselves in my work in a sense you know I I'm a great fan I love music I love the arts my dad was a vocal coach a bass player a musician a composer and so I grew up in a household that was filled with that kind of an artistic environment and so for a good fraction of my life that was sort of a hobby ask part of what I do but you may have seen I've mentioned it before that with world science festival we've been blending art and science in particular projects that I've been involved with have blended music blended staged blended physics all in sort of one interesting unified package another project that we're working on right now is this story I wrote about a boy going to the edge of a black hole Icarus at the edge of time which we collaborated with the composer Philip Glass to create a live stage presentation where we have a sequel to that that will be coming out you know as soon as the world gets back to normal if that ever happens so again that's a blending of my own interest in music and stage presentation together with the bread and butter of my professional career which is physics and calculations and things of that sort so I'd say over time the things that I do have become so enmeshed with one another that I don't even consider a distinction between what's a hobby and what I do they've all kind of blended together into into one one whole alright what else do we have here Gesell Jessel vara one says what's the biggest question facing physics in your opinion well it partly depends on what kind of physics you're talking about I of course come from the arena of fundamental physics gravity quantum mechanics things of that sort and in that domain I would say the fundamental question facing us is what is face and what is time that may sound silly right we've been formulating our theories within an ocean of space and time for centuries Isaac Newton puts forward space is the arena time is that which allows us to measure the change has occurred and the events of the universe take place within that rubric Einstein comes along and realizes well it's more complex more subtle space and time can warp and curve and in that way there are participants in the cosmological unfolding space expands time elapses at different rates if you're near the edge of a black hole if you're back here on earth but all of that however insightful it is doesn't tell us what space is doesn't tell us what time is we can measure these things with great precision but we kind of don't know what they are and progress has been made there's work and string theory that suggests that space and time may themselves be made up of more fundamental constituents and they may be constituents that are some in some sense threaded together held together by the threads of quantum entanglement these are very exciting ideas but they're tentative so an answer to your question I think the next big revolution will come when these ideas are not so tentative when we have consensus on what space is and what is with the hope that once we understand things well I mean the pattern has always been you don't understand something you work hard at it you research it you experiment you bang on it with mathematical equations you do understand it huh fantastic next step you manipulate it right quantum mechanics was just a collection of ideas that allowed us to better understand the hydrogen atom the motion of electrons back in the 1920s but now we regularly use quantum mechanics to build things the devices of everyday life rely upon quantum mechanics so the pattern is you don't understand something you work hard you do understand it you work hard again and now you can manipulate the thing that you began with so if that pattern were to hold true with space and time you don't really understand it you work hard you understand it better Einstein relativity and so forth but if we can go that next step and really fully understand it and by that means be able to manipulate space and time the way we manipulate electrons with quantum mechanics Wow what could that portend what could that mean what could that yield the ability to willfully manipulate space and time that's that's that's exciting that's what the next generation of you young physics enthusiasts you may enter that domain of being able to manipulate space and time and we'll I hope I'm around when when that development takes place na'vi ball asks hi professor how does general relativity help in the formation of the GPS Global Positioning System yeah that takes us another example right I mean I just mentioned how quantum mechanics allows you to manipulate electrons and that allows for the whole modern electronics industry relying upon quantum mechanics the GPS is an example where Einstein's general relativity has concrete applications why well how does the GPS system work right so the GPS system you have these satellites orbiting the Earth and we have these devices that can communicate with the satellites they can receive signals from the satellites and if your device receives a signal say from three of those satellites and those signals are time-stamped so you know when those signals left the satellite by virtue of the time difference of the arrival of those signals you can calculate where you must be on planet earth because you can calculate your distance from each of the satellites and that can uniquely delineate which point you are occupying on planet earth but critical to that is that the satellites are all completely synced up in terms of time that they are registering twelve moon at the same 12 moon and it must also be the case that your phone is ticking off time at a rate that allows it to stay in sync with the clocks on the satellite but bear in mind the satellites are farther away from planet Earth experiencing less of a gravitational field according to Einstein's general relativity their clocks intrinsically are ticking off time at a faster rate if you didn't take that general relativistic effect into account then all the calculations that I just spoke about check the time differences in triangulating your location would be completely off they'd be wrong the clocks on the satellite and the clocks on your iPhone would be so out of whack relative to one another that the GPS system would fail so you have to include the general relativistic corrections in order that the GPS system works and indeed that's what is done so it's a beautiful example where the general theory of relativity comes right into our pockets when we're carrying around a phone that has GPS sensitivity embedded in its technology so yeah quite wonderful to see general relativity as an applied science in that particular way okay let's see what we got going on here Wow 34:20 usually it's around four o'clock that I say I'm about to stop and then I keep on going for another hour but so we were doubt 2.71828 one eight hello professor what do you think oh no where did it go wow you know I touched this thing and it Scrolls by so fast sorry II which I guess was the number that you had there I've lost you and I'm scrolling back to try to find your question I can't find it oh no no there is way up there a weirdo asks hello professor what do you think about pure mathematics I'm in favor of pure mathematics I support pure mathematics I would vote for pure mathematics I agree with the platform of pure mathematics and what I mean by that is you know people are motivated to do work by different considerations when I was in college I was thinking about doing pure math and the reason I wound up not doing it is it sort of felt to me like the problems in pure math are the problems that we make up we envision interesting questions like how many spheres can you pack in a particular shape or what is the density of prime numbers if you look at the number line or things of that sort right and they're interesting questions but they are ultimately questions that we come up with what I found more appealing about physics is that the mathematics that you do is trying to solve problems that you don't come up with but rather that the universe comes up with the universe presents us with gravity we want to understand it the universe presents us with electrons we want to understand them so somehow it felt to me that the math is being used to answer questions that are less arbitrary that are part of how reality is constructed so that motivates me more than the questions of pure mathematics but oftentimes the questions of pure mathematics in time have proven to be relevant to physics Einstein borrowed the pure mathematical insights of Gauss and Riemann and others in order to understand curved surfaces but the surface that Einstein had in mind was face time the manifold called space time so it's really exciting when the pure mathematics unexpectedly has an impact on physics but you have to do whatever fires you up and pure mathematicians are fired up more by questions of their own making within their the universe if you will of math as opposed the universe of space-time and stuff okay Seth Catalano asked what is a good geometric intuition for what the Einstein tensor measures as the Riemann tensor measures the acceleration of the separation of two paths and the Rema and the Ricci tensor measures the change in in volume so so here's here's how I think a good way of measuring of thinking about the Riemann tensor maybe I'm gonna go back over here so starting a broadcast let me go to my iPad can it work yes it does okay so when you think about the the Riemann tensor so if you have our Alpha Beta Gamma Delta the question is what is this thing measuring and what I like to think about this measuring is it's a measure of what I have over here it's a measure of the degree of variation so let me choose a different color here so imagine you've got a shape like this and you take some tangent over here give it a different color come on hot pink is a nice one let's say you have a tangent vector over here and you slide it around to let's say you slide it around over to this location boy have a little bit of trouble here little bit of lag there we go and that's sort of path 1 motion but let's say you take that same vector and you parallel transport it around the loop in the other direction and let's say it takes you over here and in general as I was saying before on a curved surface the vectors that you get by that parallel motion will not agree what does the Riemann tensor do the Riemann tensor measures the difference in the direction that those two vectors point and that's really what it does and as we'll see when we come back to your daily equation next week I may derive it in some detail but you'll see quite directly that the Riemann tensor is nothing but the gadget that captures the difference between two vectors parallel transported opposite directions around a loop and that is the diagnostic for the surface being curved and that is indeed with Riemann tensor measures ok so what do we got do I have my iPad down I think I do tell me if I don't Agee writes AG until the end of time oh nice so you have the title of my book you wrote about the infinitesimal odds of park was arranging into orderly low and tropic States billions of people interact with reality daily so why haven't we heard or seen said phenomenon and and yeah you're right you know I I'm fond of emphasizing that the second law of thermodynamics talks about the increase of entropy but it's not a law really listen to me the second law of thermodynamics is not a law in the usual way that Newton's law is a law or Einstein's equations are a lawful description of how a physical system evolves the second law of thermodynamics speaks to an overwhelming likelihood of what will happen and the overwhelming likelihood is that entropy will increase common examples if I were to drop my iphone a knock Oh Apple product again you're going to accuse me a pot product placement so let me take something else so if I were to take this candle alright it's a glass jar if I was to drop this we know it will smash on the floor that's going from order to disorder the reverse process is possible where as the questioner asks the shards of glass on the floor in principle they can jump off the floor reassemble into this pristine glass candle the question is why haven't you ever seen that it's a very straightforward answer if you look at the number of events that all the people on planet earth right so I don't know how many people have lived since the beginning of our species I'm gonna make a wild guess let's call it I don't know 50 billion people 100 billion I don't know so there's some number you can look it up you tell me think about the number of seconds call an event sort of an a it takes one second to see an event think about the number of seconds that have elapsed since the beginning of our species that is the number of seconds that people have lived that's a number I don't know what that numbers gonna be I don't know let me call it I don't know it's called ten to the twenty you know I don't care something like that if you compare that number to the odds of the glass shards jumping off the floor because the floor boards and the molecules of air in the room all reverse their motion and reverse their members in such a way that they hit the floor boards and the floor birds jiggle in such a right way that they kicked the glass shards off my floor to reassemble into this glass the odds of that is spectacular less than ten to the minus twenty it tend to a huge number and therefore it is completely unsurprising that no one has ever seen that process happen and no one has ever seen an egg on smash no one has ever seen a candle unburned no one has ever seen a person resurrect or say evolved from an elderly physical form to a young physical form it's not that these processes can't happen they are allowed by the laws of physics it's just that the odds of them happening is not just small it's spectacularly smaller than the number of events that every human being who's ever lived has ever encountered and that's why it makes perfect sense that no one has ever known in a replicable manner has ever claimed that they have seen an entropic li decreasing process to happen that doesn't satisfy you feel free to ask again but that is what it is let's say we got mr. castle asks going to black holes can photons be trapped in in stable orbits outside of a black hole photons can be trapped in orbits around a black hole so maybe we'll do this if you want I'm going to talk about the black hole solution to Einstein's field equations of general relativity and once you have the solution you understand the geometry outside of the black hole once you understand the geometry the warps and curves in the vicinity of a black hole you can calculate how things will move how do you calculate that objects want to move along the shortest possible trajectory they're called geodesics in order to figure out the shortest possible trajectory you need to know the geometry once you know the geometry you've got everything that you need in order to figure out trajectories and indeed you can find trajectories that light will travel light itself will go into orbit around a black hole but it's not a stable orbit if you were to perturb that orbit a little bit photon goes one way or the other away from the black hole or toward the black hole so you can have photon orbits absolutely that's that's pretty cool that's pretty surprising in its own right Brad Dibble asks dr. Greene I love baseball and and physics great does the Louisville Slugger in your room any significance do I have that Slugger in here maybe it maybe it's yeah actually it is I see it in the back corner over there does it have any significance well I actually once did a physics video with the New York Mets you know baseball team New York Mets I went to them and I described how the various things like the Magnus force are responsible for why a baseball curves when a pitcher throws it appropriately I described how a baseball bat has a frequency response function that determines where the sweet spot on the bat will be when a batter is swinging that bad so I too love baseball mostly because my son loves it and he sort of wrote me back into the sport after I hadn't really thought about it for for many decades but the reason why I have the Louisville Slugger in my room here is a little less erudite than any of those descriptions might suggest and by the way you can find those mets videos those baseball mitts videos online maybe it's even on the World Science Vessel website I don't know but the reason I have the Louisville Slugger here is that I am in a little writers cabin you can sort of see I'm you know by a barn somewhat in the woods I'm I don't know about 300 feet from the main house where my family is and where I sleep I sometimes come out here at night it's kind of dark pitch dark and I'm always somewhat concerned that a coyote or a bear might want to have a conversation and route from my house to this cabin and so to dissuade certain kinds of conversations with those wild animals that I would prefer not to have I bring the back with me no doubt in a wrestling match with a coyote or a bear I think I'd lose even with the baseball bat but psychologically it makes me feel a little more confident as I cross in the darkness that is the only reason the baseball bat is here let's see Robert Taylor asks do we humans give meaning to the universe and if not what is the meaning of it all what is the meaning of it all Robert Taylor asks and yeah you know as you know you know the book I wrote recently until the end of time a little bit different from my previous books and that it takes up questions like that what is the meaning of it all and and look my view on this is is the following fundamentally there is no meaning built into the architecture of the world there is no fundamental notion of meaning to use another metaphor that's stitched into the fabric of reality at rock-bottom as I said before I believe and I think the evidence strongly supports the perspective but it doesn't prove it that reality is nothing but stuff fundamental ingredients governed by fundamental laws that we may or may not currently have at our disposal and that's it so from that perspective you can't look to physics to provide any notion of meaning or purpose meaning or purpose are concepts that really only come into existence when certain sophisticated refined collections of particles are able to form and they form by the process of natural selection that's why we are here the fundamental laws are supporting a process that Darwin was able to identify in which molecules can become ever better adapted to their environment ultimately allowing complex structures like life and like conscious beings to exist and therefore when we ask ourselves about meaning and purpose of it all we are asking a question that couldn't even be asked until self-reflective conscious beings emerged and therefore what kind of answer should we search for so we search for an answer in the fundamental equations no fundamental equations don't know about this notion of meaning and purpose where should we search for the answer we have to search within ourselves because we are the conduit by which this question is able to be asked and so ultimately it's a very personal question this way of thinking about things frees us up at least from my perspective to stop looking for some meaning out there in the world and each of us is charged with a task of manufacturing our own meaning and by the word manufacturer I don't mean to denigrate in any way shape or form the concept of or purpose or meaning not at all I think that the fact that we manufacture it makes it exquisite it grand eise's it because unlike other approaches that may look out to some being or look out to something in the world to bestow meaning upon us that version we're just wearing someone else's notion of value in person some external forces conception of value and purpose it is so much more noble to come up with our own value our own purpose and that's something that Robert Taylor you need to I knew I everyone needs to do it on their own they look for mine my perspective understanding physics understanding the universe that's part of where my value and purpose comes into the story having these conversations spreading insight that physics has revealed that gives me a sense of a value and purpose as well revealing some fundamental quality of reality however minor it might be on those occasions I have felt enhanced value and purpose through that achievement these are the things that we use and of course my family right my wife my kids that is a source of great value and purpose as well so you won't find it written into the Stars you won't find it written into the molecules you'll find it if you allow yourself the freedom to make it up find what gives you that sense of purpose and value and that is a highly personal highly subjective but deeply exquisite way of addressing the question of meaning I mean if you guys feel differently than that no doubt some of you have a radically different perspective put it in the comments I'd be more than happy to hear different views all right carry on with the more physics oriented questions that I see coming first SCI asks psi I guess our side the Greek letter SCI my question is to dr. green yes they're all coming to me unfiltered will matter decay in the future will protons decay and if they will does that mean the time stops will time come to an end so we we do not know if protons will decay we believe that every other particle except the lightest ones like electrons neutrinos and photons the belief is that they are likely stable but conglomerates right now a proton is three quarks the conglomerates we believe will all fall apart but we don't know that for sure neutrons certainly fall apart short period of time hold the neutron in your hand you know a bunch of them in your hand after you know a short period of time ten minutes you know half of them will have disintegrated protons we have tested we've looked at large huge tanks of water and other fluids looking for one measly proton to fall apart we've never measured it our mathematics suggests that protons will fall apart different theories give different timescales ten to the 35 ten to the thirty eight years is the half-life for a proton but as yet we don't know but I would say that most physicists anticipate that protons are not forever that they will also fall apart now the question is when they all do fall apart does that mean time stopped since people use that language when all complex matter has disintegrated all stars have disintegrated into a spray of particles is it all gone in the sense that time doesn't take forward well time may end in the sense that there won't be any meaningful notion of change if all you've got it particles floating through the void then it kind of doesn't matter whether one particles here and one particles there or one particles move so in that sense there won't be anything happening and if time is a measure of change and change has to rely on things happening then yeah in a sense and a poetic sense timeless stopped but particles will still be moving and they're moving through time and so in that sense time may go on forever even though there won't be anything interesting happening now again if you're interested in this in more detail chapter 10 of my book until the end of time takes this on and notes that in the very far future if you wait like 10 to the 10 to the sixty-eight years then even these particles floating through the void can on occasion randomly coalesce into interesting structures structures that in fact may be able to think - Kaja Tate that's weird that's curious wait ten to the ten to the sixty eight years and a brain a human brain may spontaneously form out there in the boy towns nutty maybe nutty we generally use this as a diagnostic tool to try to modify our theories to not allow this process to happen these are called Boltzmann brains after Ludwig Boltzmann so if you're interested in the idea of both membranes again you can reach after 10 up until the end of time but barring those kind of rare occurrences that may happen in the far future the absence of change in some poetic sense can be interpreted as the absence of time so yes in short to that question a campus crypt I'd I'm reading until the end of time and almost every footnote redirects to chapter 10 that's not quite you it's not that every footnote redirects to chapter 10 but chapter 10 is where it kind of all comes together where I lay out the very very far future and in a book that's called until the end of time the chapter that takes you toward the end chime is end of time is going to be a pretty pretty vital one and that's why chapter 10 has the prominence that it does Demond Keter Paul asked as a spring gain mass when it's compressed does potential energy convert to mass and the answer is heck yeah so if you have a spring and you compress it right and you were to weigh that spring on an incredibly precise scale and compare the reading when the spring is in its relaxed ordinary state the reading on the scale will be bigger when the spring is compressed the energy that you push into it as you say it's this potential energy but Einsteins e equals mc-squared takes into account all energy it's really saying that energy and mass are different labels for the same kind of thing and so by putting more energy in you've put more mass in and that mass would be reflected on the reading of a scale similarly as I'm fond of using as an example if I have a flashlight and I put the flashlight on a spectacular sensitive scale and I turn the flashlight on or the torch as some of you like to use the language as the light races away from the flashlight it takes away energy right light has energy it's taking away energy from the flashlight taking away energy means you're taking away mass which means the reading on this fantastically sensitive scale will go down if you keep the button in the on position for the flashlight tee-ball ask dear professor green greetings from god ran away don't do that I'm gonna try to find that question I should never touch the the pad here when I'm in the middle of asking a question that's why this happens to me I'm trying to find your question I'm scroll up maybe ask it again I can't see it unfortunately so I'm going to move on to another question sorry about that Tybalt mr. castle asks dear sir other equations that you don't understand it depends what you mean by understand you know I feel like I understand the the key equations of modern physics in the sense that I know where they came from I know what they're trying to tell us but do I understand them deeply intuitively in my bones some of them yes some of them no I often find that I have to go back and rethink things especially when I'm teaching a subject even if I think and I understand the subject inside out I'll come upon some equations say wait a second wait what is that really telling me and where does that really come from and what's the chain of reasoning that gets me to that equation I find that I have to go back and refresh myself in order to keep the chain of reasoning really sharp inside my brain so you should never be what's the word you should never be what's the words not disgruntled distressed it's a very basic word and you should never feel bad you know tell me what word I'm looking for if you if you sort of find that you have to go back and relearn something from the past I've got to do it all the time so in that sense I have to refresh my understanding but there certainly are fields of mathematics that I've given up on somebody asked earlier about math that I was surprised I had a role in physics and I described algebraic geometry as an example of a field that unexpectedly I had to learn parts of it to do certain research in string theory but there are other parts of algebraic geometry you know in the book of joel harris has a beautiful book on algebraic geometry and there are chapters in that book that defeated me I wanted to understand those chapters and I never felt like I really did and their other chapters that I didn't even bother to try to read so in that sense there are equations in pure mathematics that I feel stretched my brain to the limit and could I have learned them if I if I really really really tried maybe but it was the getting to a point in certain of those undertakings that I felt like every time I went further in understanding part of the subject another part of it would sort of disappear as I was like a conservation of the amount of brain space that my brain was willing to allow the equations of algebraic geometry to fully inhabit so yes they're definitely things that I've tried to understand and felt like I didn't fully master or felt like I had reached my my limit okay Ludwig von Beethoven asked will you write not a biography someday in the distant future great source of inspiration to me and many others who I'll be honored to be enriched by your life experience von Beethoven um I don't know um I'm not sure it's never really occurred to me but you know in in my latest book there is much more personal reflection than I've had in any previous book and and some people have asked me why not in a in a accusatory way but they were just curious about my own thought process that led me to include more personal reflections in until the end of time let me tell you why the point of the book until the end of time is that there's not one way of understanding reality we physicists focus on the fundamental particles and the fundamental laws and that's what we're really talking about here today but to have a full account of the world as experienced by a human being you need to layer various other accounts into that story you need to layer the account that the chemist gives at the biologist gives but you can't stop there you need to give the account that the philosopher gives or the neuroscientist gives or the artist gives or the theologian gives and all of these accounts serve to enrich your understanding of what life and what reality what the world is all about and so it felt to me that it would really be unfortunate in a book that's talking about these layered account these layered stories to not include the only human story that I have direct access to which is my own so at various moments you will see that I do layer in personal experience and personal reflection as part of that nested account so that the narrative can be as rich as possible but rather I'll go further one day and and actually write an autobiography I I don't know I don't know we'll see we'll see how things progress okay megha asks if all information is stored in quantum entanglement doesn't that mean eternal life actually happens and I would say no not at all so you can have a system whose information content is fully stored in the quantum state of that system and if you then conclude or at least hypothesize and it seems to be well bolstered by our understanding of physics that information is is not destroyed that the information is always there at best what I'd be willing to say is that in principle all the information contained in a given human brain that information can in principle be recreated by infinitely precise measurements that take place after that being may have expired but at best you can imagine using that information to recreate the original living system it's not the developing system itself is eternal you know it's hard to define what you actually mean by death that's actually reading a book reasoning by a philosopher who is you know struggling to give a precise definition of what it means when we're when we die but I'm I'm pretty satisfied with the straightforward definition you know when this physical structure called Brian green and when it no longer functions when it no longer carries out the basic processes that we define as living when no longer breeze or eats or thinks that's to me when it has died and when it dies sure in principle the information content and all the constituents and their various interrelationships could in principle be recorded and could in principle be recreated but still I'd be dead there could be a recreation of me perhaps maybe we'll get to that point maybe you know in the sci-fi books we imagine uploading ourselves into a different physical structure will will that ever happen I have no idea but even if it does happen it's not to say that this living system that originated the thoughts and feelings and emotions and memories one have died it will have died will simply have found a way of capturing its patterns and allowing this patterns either to be recreated or to persist that's that's different from saying that the living system itself is eternal oh yes Joe Harris is the other author as Evy notes to me on on on that book so Griffiths and Harris yes its Griffiths and Harris that's what it is exactly right thank you let's see what else we have somebody is directing me to a question on Twitter but I don't know what question that is sorry let's see so I am I like that I am asked what is the difference between gravity and quantum gravity and why are we after the latter why are we after quantum gravity so so it's it's quite straightforward I am and and the answer simply this so in the early days of trying to understand reality Isaac Newton gave us the universal law of gravity that perhaps you know about F equals G m1 m2 over R squared we call that a classical formula because it does not take into account the realizations that we acquired in the 1920s and 1930s that the world's not actually governed by the classical laws of Newton there's another class of laws the quantum laws and we've been very good at applying those quantum laws and situations of individual particles collections of particles molecules and atoms and so forth the equation that we make use of is called Schrodinger's equation now although we understand gravity in the macroscopic way that it manifests itself a la Newton in F equals G m1 m2 over R squared there are situations in which Newton gravity and quantum small things need to be blended together so a quantum theory of gravity is a theory of gravity that will work not just for stars and planets but will work for individual particles like electrons and quarks and protons and moreover it will work in any situation where a quantum description of the ingredients is necessary so a quantum theory of gravity blends the rules of quantum mechanics with the rules of gravity in a manner that allows us to apply the gravitational force in any circumstance not just planets and galaxies when things are big but also to the realm of small so being even more precise Einstein gave us a more more refined classical theory of gravity the general theory of relativity is a classical theory it doesn't speak to quantum physics so our goal has been to blend in Stein's theory of gravity the theory if you will of big things with quantum mechanics our theory of small things so that we have one unified description that works for the small works for the big works for everything in between right now we have a fractured description we use quantum mechanics for the small Newton or general relativity for the big depending on the circumstance we don't want two theories there's one universe there should be one theory that puts it all together that's the goal of a quantum theory of gravity all right so let's see what we have you sir on your book fabric of the cosmos you have mentioned that space and time are moving equal to the speed of light please explain to me as one of your readers so I think what Safi cate rasa is referring to suffocate suffocate rozzers referring to is that any individual can be thought of as though they are moving through space-time at the speed of light what do I mean by that well for light it's obvious light just moves at the speed of light but what about me right here imagine that from my frame of reference I'm not moving through space at all am i executing some motion and the answer is I am I'm moving through time right my clock is ticking off second after second after second so if we don't just think about motion through space but also think about through time that I'm moving right now I'm moving through time at what speed am i moving through time and it turns out that one way of interpreting on some special relativity is that I'm moving through time at the speed of light and in fact any motion at all when thought about not just as motion through space but as motion through space-time has a speed equal to the speed of light why is that useful that's useful for the following reason if I now get up and start to move through space according to Einstein Einstein doesn't say it this way this is sort of my interpretation and there are others who say this too but but I kind of saw myself writing this interpretation down the first time in the elegant universe it was but whatever who cares about credit or anything like that but when I get up and start to move through space I divert some of my previous motion through time into that motion through space and therefore my passage through time and motion through time slows down when I get up to move through space it affects my passage through time that's why if I look at you getting up and walking and I look at your wristwatch I will see your wants ticking off time slowly why the previous motion that you had was all through time you got up and you have a fixed amount of it you're moving through at a fixed speed and therefore if you get up you divert some of that fixed speed from the previous motion through time into your motion through space and therefore your motion through time slows therefore your clock ticks off time more slowly that's one way of thinking about why it is that time slows down for an object that you are looking at in motion why does it slow down because as a fixed amount of motion through space-time and it is diverting some of that motion into motion through space therefore the previous motion through time must give up some of its speed it must slow down clocks must tick at a slower rate that to me is the most beautiful most intuitive understanding of why it is that when an object moves time slows down Pericles Mello asks do you have a video on the double slit experiment that shows that and observe it behind this that collapses the wavefunction and the pattern of superposition on the screen I don't have a video of it but I'm wondering Pericles Melo why you're asking for one because your description is pretty darn good right in your mind's eye you can picture no observer you got the particles going through the two slits you get the interference pattern light dark light dark light dark on the screen if you then modify the experiment by putting an observer on the other side of the barrier who measures the slit through which a particle goes that interaction changes the experiment has it changed it the particles will no longer land in this nice light dark light dark spread instead they will fan out and just give us an overall smear on that back screen no interference pattern at all so your description captures it right so I think your mental image is just as good as any video image that I could give you amanda says time has to compromise for space yeah that's right Here I am my watch is ticking off time all right I get up and I start to walk and you will see time compromising for space-time is giving up some of its fixed light speed motion so that some of that motion can be diverted into my motion through space that's it that's really what Einstein found in the special theory of relativity totally intuitive when you think about it that way however strange the implication may be okay oh okay t-bolt has reacted question thank you and I'm not going to touch so I'll read a dear professor Greene a recent became a big fan of yours exclamation point thank you my question is or was why our past present and future and illusion thank you very much so so teapot you're referring to a quote of Einstein's a famous quote it was a quote that Einstein wrote to the widow of his friend the Bissell and what Einstein said to be Tso's Widow was the fact that he has departed the world a little before me has no meaning because the distinction between past present and future is only an illusion however persistent that's the translation that I prefer of that quote I think it's a quite beautiful quite poetic quote how do you interpret that physically what was Einstein thinking well look you know he was writing to the widow of a friend so trying to overlay a precise interpretation might seem a little crass but let me give you the interpretation that I think works best so in the special theory of relativity we learned that our clocks tick off time at different rates if we are moving relative to each other what does that mean that means that what you consider to be right now may not be what I consider to be right now you're now maybe what I consider past you're now maybe what i consider future and similarly for your consideration of my notion of now which means if you consider everybody's perspective imagine that they're our perspective all throughout the cosmos even hypothetical perspectives all moving relative to one another and so forth and if you take into account every individual's notion of now then you basically will span all moments of time moments that I consider to be way in the past someone way out there will say no no that happened just now notions that I say haven't yet occurred are in my future someone way out there say no no that occurred just now and therefore the distinction that I make between past present and future is not a distinction that holds for all the individuals out there so at the level of the individual there is past present and future for sure if you lift yourself out of the perspective of the individual and consider a more global perspective there is no organization to past present and future that works globally it's all individualistic it's all subjective and therefore since it's subjective Einstein is saying in that sense it's an illusion it is not woven into the fundamental fabric of the world this distinction meaning past present and future in that sense it is an illusion ok what else do we have here what time is it oh it's it's 5 o'clock but if you still want to chat a little bit further I'm I'm happy to do that it's uh it's 236 in India you all should sleep now one of the viewer says some viewer who's called viewer of all things could a new universe spontaneously come into existence when another universe has expanded to emptiness and and and yes I mean it's conceivable that the Big Bang is an event that can happen arbitrarily many times the required conditions for a Big Bang is to have enough uniform energy in a small enough region of space it's usually carried by what's called the Implanon field in the conventional mathematical description and that then yields this repulsive force that blows space to swell enormous Li and in principle quantum mechanics allows you to calculate the likelihood that such a uniform sufficiently dense region of inflict on energy could form at any given moment and the answer for that is a small but nonzero number so if you wait sufficiently long yes you can imagine whole big bangs happening long after our universe has expanded in the language of the questioner expanded into what is as close to nothing as you could imagine ok what else do we have unorthodox what do you think of flat earthers well you know there are truths about the world that are objective there are truths about the world that are subjective that speak more to our place in the world as opposed to qualities of the world that are accessible to anybody the earth in the objective sense is not flat the earth in any objective notion of truth is not flat but if someone uses the notion that earth is flat as a metaphor that helps them to better appreciate where they fit in to the reality that they inhabit and so bit but goodness gracious keep straight your metaphors and your reality there's poetry and there are facts and the fact of the matter is that the world has a particular objective shape it is a sphere as close to a sphere it is not flat but I'm all too happy in poetry to hear about Flat Earth in economics to hear about flattening the disparities among different places on earth but that's metaphor keep your metaphors straight keep your poetry stay keep it all beautiful and wonderful and insightful and subjective just don't mistake it for objective truth okay what do we have here Oh t-bone thank you very much thank you I appreciate your response to my answer what else do we have here why does hackers lab asks why does string theory require 11 dimensions of space and the extra dimensions of strength their particular number is one that I have tried ever since writing the elegant universe to find some non-technical way of explaining that and I really failed on that I don't I've never seen a non-technical explanation for why string theory requires extra dimension so let me give you the semi technical explanation which is the best that I can do there is an equation within string theory which must vanish for the theory to be consistent all right meant that already is technical from the get-go so I apologize for that that equation takes the form it turns out a complicated expression times something really simple so in the simplest case it's just d minus 10 where D is the number of space-time dimensions in string theory and so for this to vanish the complicated thing all it doesn't vanish the simple thing must so that you have something times zero how does D minus ten equals zero D equals ten I swear to you that's really what happens it's a calculation in perturbative string theory they get two to ten as a number of space-time dimensions to get to 11 you then realize that that perturbative calculation leaves out one dimension this is what Edie Witten alerted us to in a beautiful lecture at University of Southern California in 1995 and and so we've pieced together our understanding of eleven space-time dimensions but I wish I could just say to you hey you know just think of a marble on a trampoline and then give you a few more words and we get to the extra dimensions I've never succeeded in doing that I've never seen anyone who has so unfortunately I can't give you a nice non-technical explanation for the extra de Levin extra dimensions now Joelle jarvan then is making good reference that it goes to 11 no doubt that is a spinal tap reference if I understand Joel correctly where one of the musicians is so proud of his new amplifier because whereas most amplifiers have a top level of 10 his amplifier has a top level of 11 so so if that if that explanation gives you any insight fantastic for me it doesn't do much beyond make me smile but that there's nothing wrong with smiling aya Hasson asked what is the use of string theory what is the use of string theory well the use of string theory is number one it is our most promising theoretical approach for lending quantum mechanics in the general theory of relativity so there it is right within string theory the mathematics makes clear that quantum mechanics and gravity can harmoniously co-exist this is a big problem a big challenge what else does string theory do well it shows us at least in principle how basically every discovery from the past 50 years in physics finds a natural home within the mathematical structure of string theory the Higgs particle it can fit engaged symmetries that we use to describe the electromagnetic and the nuclear forces they have a natural home the discrete symmetries that protect certain properties of particles like stability of the proton base of a natural home within strengths eon and on string theory not only puts gravity and quantum mechanics together but it allows all of the major discoveries of the past to also be brought within the fold string theory is also valuable as a tool other subjects have found that string theory allows you to make calculations that would be horrific without the mathematical tool of string theory so there's a great deal of use of string theory in the physical sciences but the real question is is string theory the right way to go if you want to find the deepest laws of the particles and the deepest laws that govern them and on that we don't know we are still struggling to connect string theory with observations in a manner that would address that question so there's a lot of use of string theory a lot a mathematical user strengths here I should have mentioned that some of my own work open the door for string theory solving certain math problems that mathematicians were stumped by and string theory can answer those questions so on the math front on it's applications and other fields and potentially as a unified theory of all forces and all matter so why do we work on it we work on it for that reason the potential is enormous when will we know if string theory is Right theory through observation and experiment I don't have a crystal ball I don't know it'd be great if it was soon but the world universe doesn't look to satisfy our timetable the universe will reveal its secrets when we have the capacity to interrogate it in the right way with the right equipment so Damon Keter Paul asks is it resolved that free will does not exist any logic in favor of free will any any good reads on the subject well Damon I'll refer you to my own thinking on this I'll briefly describe it but Chapter five of my new book until the end of time it's called something like particles and consciousness and it comes to the conclusion that there is no free will at least in the traditional sense of the term the sense that we all have that we are the ultimate originators of our actions we are the full author of the things that we do that's sensation that intuition runs into problems it runs into serious problems because when you recognize and not everybody agrees with this I mean there is a review of my book that came out just the other where the reviewer clearly does not buy into one of the key assumptions that I make which is fundamentally the world physically is made of particles governed by laws you don't need anything else beyond that to explain the phenomenon of existence now is that absolutely true I think it is many others I'm not alone I'm not trying to say this is an original idea nor am i trying to point to others to bolster my position but the fact of the matter is that many people who have thought hard about this have come to the same conclusion there is no life force out there there is no consciousness field that we somehow tap into the world is made of ingredients governed by laws and once you accept that and you don't have to I don't need you to accept that I'm simply giving you a conclusion based on that way of looking at the world once you accept that then free will goes out the window because we as collections of particles are governed by mathematical law and the motion of our particles therefore is not under our control and my dog Luna is clearly agreeing with me one more time Luna let's hear you do you agree the dog community agrees with no free will and that's just being silent but once you recognize that your particles are under marching orders dictated by mathematical decree and once you recognize that every choice you make every decision you make is nothing but your particle is moving one way or another inside your brain or your body's particles moving one way or another you don't have any choice in how those particles move those particles are fully governed by quantum mechanical marching orders and in that sense the traditional notion that you can intercede makes no sense it's not as I've said before as if the laws of physics are constructed in a way say for me that they say hey particle is making a Brian green follow Schrodinger's equation until Bryan's about to make a decision then hold wait for him to make a decision and once he makes a decision carry out the decision that he made no there's no place in the fundamental laws for me to have any role like that whatsoever so in that sense the intuitive notion that we are these agents volitional agents that have choices and decisions at our disposal in that sense it's wrong the decisions happen and we recognize when they happen but we in some sense are pulled along by the very same laws that pull along our particles now again are there other perspectives absolutely and in my book I give some of the other perspectives I don't think they're right and yet there are many opinions and as yet we don't have enough data to fully articulate an unassailable argument that would establish the position that I just laid out to be true but I strongly suspect that it is true okay what else do we have here give one that has not asked a question before Bob g'day asks dr. green are quantum fields the basic building blocks of the universe well our most refined and tested description of the world is in terms of quantum field theory so it is fields that are governed by the generalization of Schrodinger's equation from particles to fields it's the same equation just now apply to the ingredient of fields and fields are able to create particles so fields are able to embrace both the more spread-out quality of a field like you know an electromagnetic field light as well spread out but the mathematics is simultaneously able to describe the more particulate entities the small excitations of a field so if you were to ask me what is the fundamental description of the world that has achieved a level of consensus because of observation experiment then I would agree that it is quantum field theory and it may be the quantum field theory is the be-all and end-all in fact there's evidence that string theory is equivalent to quantum field theory just sort of rearranges the calculations and the ingredients in a different way so in that sense even string theory may be nothing more than quantum field theory so the answer to your question is it might be yes humility suggests strongly that we should be tentative and say this is our current level of understanding it could change 5 10 100 years from now but yes based on our current level of understanding that's where we are all right let's see let's see I wonder if we should start to wrap this up as I'm starting to lose my voice but let's just see what else comes in and keep forgetting to look at the Twitter list but it doesn't seem to have been updated or have a Facebook lift so Cova smiley from Facebook asks if gravity can bend light how can it not slow light down and that is a very very good question and here's the answer so when light travels from our macroscopic perspective we might see it traveling on some curved trajectory and it's going by the Sun we say the bund the Sun bends the light bends the trajectory of the light but if we now put ourselves actually in the shoes of the photon itself the light itself what it actually is doing it's going on the straightest trajectory available to us it goes along a geodesic in the environment so the macroscopic global view of this curved trajectory is actually nothing but the photon traveling along the straightest path available to it the shortest rejector you sort of familiar with this you know on earth when you get on an airplane and you go over a long distance sometimes the trajectory takes you over the North Pole or something you're like what in the world why would we be traveling along that trajectory you're your brain and your image of a flat map would suggest a different path but actually the shortest path the straightest possible path on a sphere is a great circle an arc of a great circle and sometimes those arcs pass over the North Pole so the light itself is not actually from its perspective going and a bendy way at all locally it's going on the straightest possible path so its speed locally is always equal to the speed light is always going in a nice straight trajectory it's just when you piece together all those straight trajectories from a global perspective it looks like it went on a bent trajectory but velocity is a local notion it's what is its velocity at a given moment at a given location it's always the speed of light and it's always traveling in a straight line okay what else I do it that was a good question on Facebook maybe another one on Facebook some there there long to read so I'm going to go back to the shorter one on on YouTube reckless abandon asks about background know maybe recklessly Ben is having a side conversation with Konstantinos panagiotis talking about background independence the electrode by Smolin and and rivaly so me let me talk about background independence since you guys are having out your passing notes to each other behind my back so I'm gonna jump into this background independence conversation and this is a conversation I've had with Lee Smolin many times and I think we see eye to eye on it so loop quantum gravity from the get-go has this quality of background independence which means that the the equations are not tied to any given shape for space-time they're agnostic with respect to the shape of space-time and that's a good thing because ideally you don't want to put space-time into your theory you want it to emerge from your theory and loop quantum gravity promises that from the get-go in string theory the way we have developed the theory is in what's known as perturbation theory and that's now we've gone beyond that with certain developments in the last decade or so but let me just go back to the beginning as it's easier to describe so we initially described string theory in terms of small fluctuations about a given background and therefore the claim was made that string theory is not background independence you sort of got to give the shape of space-time and then gravity amounts to little tiny ripples imposed on that pre-existing shape for space-time so there was a conversation that was happening for a while that loop quantum gravity is background independent and string theory is not background independent but the fact of the matter is in our understanding of string theory we can arbitrarily perturb that background get to a different shape and then talk about graviton zazz little fluctuations about that new background so what I like to say is string theory in that formulation doesn't have manifest background independence there is a background in our description but the equations are sufficiently general that they are amenable to an arbitrary change of that background so while they don't have manifest background independence they do have as much background independence as they possibly could given that that perturbation theory needs you to specify a background around which to perturb developments more recently that have given us more non perturbative formulations the ADF C of T correspondence allows us to speak in a manner that's more general than the one that I just gave but the conclusion is the same string theory is as background independent as it possibly could be expected to be okay hopefully that helped in clarifying that side conversation that you all were having but through what we have here pure asks your thoughts on the graviton you know does it exist I think it probably does exist I don't know if we're ever going to detect an individual graviton again gravity is an incredibly weak force it is depending on you measure it something like 40 orders of magnitude weaker than the electromagnetic force if I had say two electrons repelling each other the strength of that electromagnetic repulsion would be 40 orders of magnitude or sir larger than their gravitational attraction gravity is just really weak and a graviton is the smallest packet the smallest bundle of the spectacularly weak force so to find the smallest part of the weakest force is a huge experimental challenge and not one that I anticipate anytime soon that we will overcome but do I think that the graviton is real yeah I do okay what else do we have here um reckless abandon came back at me that's what was fun but now that ran away I think I know what it says I think the question was doesn't the background that you select around which you perturb doesn't that define a vacuum stage yeah it does so for any given geometrical shape there is a vacuum state my point is at string theory the equations are not really tied in any fundamental way to that shape because you are able to perturb arbitrarily far away from that shape you have the same darn equations then you can consider small fluctuations say to talk about a graviton about that new vacuum state so the dependence on the background is not a string theory dependence per se it's a perturbation theory dependence it's the way you're analyzing the theory as opposed to an intrinsic limitation of the theory that's the point that I am making our patek says can you finish the QA because I need to go to sleep how about I will tell you Arpit tech I give you permission to go to sleep you're not gonna miss anything well you may you may MIT you know what on second thought are putting up if you want to really hear everything that's gonna happen here you better stay awake for just another few minutes I'm gonna I'm gonna wrap this up pretty soon I always like to wrap it up on up on a high note some question that sort of blows me away and let's see what we can do to get to that well here's a here's a good one a frequent one but Fatima Syed Hasan asks where does the universe expand and I presume what that means is sort of what does universe expand into again it's a natural way of thinking about things based upon everyday experience man it's getting pretty sunny in here boy and I don't have any window shades but whatever so I probably look like I'm in the witness protection program probably half of my face is bright and a half is dark but the when you think about anything expanding right when you think about a balloon expanding when you think about bread expanding it's always expanding into a pre-existing space and therefore you naturally imagine that when the universe expands it too must expand into a pre-existing space sort of some some real estate that the universe has somehow not yet occupied not yet claimed hegemony over it must expand into that and that may not be right in the traditional way of thinking about the expanding universe is not right we imagine that as the universe expands it creates the very space that it expands into that space wasn't there before the expansion the expansion is literally the creation of new space new territory and the universe inhabits it by virtue the fact that that territory came into being because the universe expanded it's not that expanded into it it expanded by virtue of creating it that's that's the difference in thinking about the expansion of bread and the expansion of the universe okay what else do we have enough Abu Salim asks could the universe have begun as a black hole yeah I mean Lee Smolin was mentioned earlier and and Lee has an interesting theory that black holes give rise to other universes in some sense the interior of a black hole opens up into a new realm and he has an interesting notion of natural selection among universes namely that those universes that have constants changed in just the right way to maximize the production of black holes thereby producing more universes more children more progeny are the ones that are better adapted and therefore will dominate reality so he makes a prediction that the typical universe in this ensemble should be one whose parameters masses of particle strengths of forces are finely tuned to maximize the production of black holes you know you want to have a lot of kids in order that you dominate the Darwinian evolution the universe wants to have a lot of extra universes produced in order that it dominates that competition and the way it dominates it in this perspective is by having a lot of black holes so yes it's possible I don't know I've never been convinced of that idea it's an interesting it's very creative so I'd like the idea a lot but I am not convinced that there's any real data that allows us to fully talk about how a universe would emerge from a black hole nor how the constants of nature would change as you pass through a black hole a lot of details that have yet to be nailed down before I would take that picture seriously ok so what do we got Andre and I'm drew Canole Sookie how can one get into string theory after grad school I would say get into screen string theory during grad school if you could I mean I think that's a that's a good way of going about it past grad school what can you do sure I mean people change fields all the time getting into string theory is more of a challenge because it's the most highly mathematical of all the theoretical pursuits that you might have undertaken and that means they'll have to learn more and more stuff that you could have learned as a grad student but you probably didn't if you were studying a different field not impossible to do can be can be difficult to do and ruhani asks what is energy and multiplier fan says do you love me not your which answer which question to answer first so I'm gonna I think I'll just answer one of those what is energy so you know energy is a very subtle idea we throw the word energy around all the time in in in basic physics class you learn energy is anything that has the capacity to do work work is moving an object through a distance right so I'm exerting energy to move an object through a distance a force exerted through a distance and um I find that's not a bad way of thinking about things but I think having an inventory of energy gives you a feel for it so mass is a kind of energy motion is a kind of energy kinetic energy is a kind of energy compress a spring or stretch of spring potential energy is a kind of energy energy from gravitational potential you see its energy can be turned into into motion so once you sort of having your disposal some of the examples of energy now you can say energy is anything that I can acquire from those ingredients through an allowed physical transformation potential energy to kinetic energy nuclear situations where we can take energy that's strong that's in a strong nuclear force and turn that into kinetic energy so it's basically anything that has the capacity to impart motion okay what else do we have here Stosh asks Stosh I like your question genetic plasma nuclear energy all types of energy okay thank you piping in on that PHR asks maybe this is a good one to end on piata asks what is life and and again it's a question that you can gain insight into by taxonomy by giving a listing of examples of life well why am i hearing myself talking oh I'm having maybe this is a sign that I should go up on self-talk what it was yeah what did I do wrong did I hit a button can i moot myself or should I sign off is this the sign from the gods that I should give up feedback anymore alright I think we're good do you guys still hear me out there in the in the YouTube world or am I now talking into the void let me know if you can still hear me and also let me know if you want me to to wrap this up I it's uh it's 5:39 I figured I could go for a few more minutes if you want it's it's still light out here and I'm not gonna do this in total darkness although I do have some lights on in here too but what was I talking about Oh what is life let's see where that takes us so there's some examples of life right human being dogs cats bees insects birds dinosaurs right so having the taxonomy and mine is useful to thinking about life but it's notoriously difficult to give an ironclad definition of life I think the definition that was on one of the spacecraft that had near the record the recording of sounds of planet Earth described life as I think anything that had the capacity to carry out Darwinian evolution by natural selection I don't know I don't that really helps you get a feel for life but life is anything that can carry out the processes of life is kind of circular but life is anything that can move it's something that can metabolize right it can take an ingredients from the world and turn them into energy to power internal physical processes so you know there all sorts of criterion that you might put out for the Bennett man there's all sorts of stuff having that is an alert happening on my phone it's a tornado coming seriously you guys see this I may have to uh I may have to sign off if the tornado comes yeah this guy is turning kind of dark but you know I think I'm in a relative safe spot but anyway if you see me swept away by a tornado you know that I have gone to the Land of Oz or something of that sort but yeah the the forces are seeming to tell us to wrap it up but I'm gonna those are wrong but my point is there are many boxes that you can tick such that a living system satisfies the criteria in your list but I really do think that life is a continuum it's not that life is something that satisfies this list of criterion rather if it does satisfy this list of criteria then you can call it life but if it doesn't it doesn't mean that it's not life it might mean that it is enroute to the kind of life forms that we are more familiar with but I think that life extends all the way back and at some point in the Darwinian evolution you get to a level of complexity and refinement where it's obviously a living system but I don't think there's a place where you can say that life turns on life is organized complexity life I like to say is physics orchestrated and if that orchestration has sufficient robustness and complexity then you have a living system and when it doesn't quite have that you are enroute and perhaps better thought of as being along the spectrum that goes from inanimate to animate not something that necessarily turns on like a light switch all right guys um the wind is really picking up here and I see all my family heading inside and they're probably going to bar the doors if I don't get in there in time so I think I'll probably wrap it up here and that's just like one more question that I just have to answer and I'm scrolling through Oh Kent Linkletter as is a Boltzmann brain alive well yeah I would say that in relation to the question that we just had if you had a Boltzmann brain out there in the void able to carry out even momentarily the processes that this great thing inside of our head does and if that brain you know has the thoughts that for instance match my own if it's particulate arrangement happens to mimic and replicate the particle arrangement in my head right now I think I would absolutely call that Boltzmann brain alive and after all if I was able to grab that brain and stick it inside a human body and connect it up appropriately that brain would function precisely as an ordinary human brain so I would certainly characterize it as a living system it might not live long without the life-support that a body and a planet can't provide but during the moment of consultation that that brain carries out yeah I would say it is a living system ok so people are telling me to stay safe I appreciate that please go to the shelter I'm gonna do that alright guys so this has been fun you know a little a little shorter then then perhaps we would have if the situation hadn't changed next week maybe we'll go for an even longer marathon session until then you know watch until the non till the end of done it's my book I mean take a look at your daily equation videos catch up on any that you've missed let me know I swear I don't know the level of rigor that you want because sometimes I'm more technical than other times and it seems like less people are engaging with those but maybe it's just a small core of you die hard mathematically rigorous people that's a fine thing too I don't need to reach a lot of people with these as long as I'm reaching people for whom they matter but I'm gonna have a blend you know of general ones and and more specific ones as I've said always I'm not sure how long I'm gonna carry on doing the series I actually have to teach a course at Columbia starting May 26 8 hey any of you you're allowed to sign up for the course unfortunately it is pretty darn expensive it's what you know universities do they have a lot of expenses so it's not cheap to sign up for my course but it is on my book until the end of time it's called origins and meaning you can search onto clumb columbia summer school if you want to do that I'd love to see some of you in that but again unfortunately I've got no control of this it does cost a little bit of money or a lot a bit of money to do that once that course begins it may be harder for me to do your daily equations so toward the end of May I may look to wind this down maybe I'll call your weekly equation if you don't mind maybe do some videos once a week and some live sessions once a week that's a possibility but you know go check it out check out the Columbia thing if you're interested check out your daily equation if this is new to you this is your first engagement with this and until my next video I hope to do it on Monday what am I gonna do I don't know I think it may do something in cosmology maybe I do the Big Bang or maybe I'll do dark energy that'd be fun to do tell me what you like me to do on Monday if I see something in your list I will do my best to accommodate it okay
Info
Channel: World Science Festival
Views: 144,868
Rating: 4.7208123 out of 5
Keywords: brian greene, Live stream, #livestreaming, #livestream, #live, #streaming, Albert Einstein, daily math lesson, Your Daily Equation, brian greene interview, brian greene until the end of time, professor brian greene, astrophysicist brian greene, professor brian greene interview, physicist brian greene, brian greene book, professor brian greene until the end of time, brian greene physics, brian greene joe rogan, brian greene string theory, number theory, daily series, math series
Id: iGDNzW59E3M
Channel Id: undefined
Length: 163min 32sec (9812 seconds)
Published: Fri May 15 2020
Related Videos
Note
Please note that this website is currently a work in progress! Lots of interesting data and statistics to come.